SlideShare a Scribd company logo
1 of 127
Author(s): Rebecca W. Van Dyke, M.D., 2012

License: Unless otherwise noted, this material is made available under the terms
of the Creative Commons Attribution – Share Alike 3.0 License:
http://creativecommons.org/licenses/by-sa/3.0/

We have reviewed this material in accordance with U.S. Copyright Law and have tried to maximize your ability to use,
share, and adapt it. The citation key on the following slide provides information about how you may share and adapt this
material.

Copyright holders of content included in this material should contact open.michigan@umich.edu with any questions,
corrections, or clarification regarding the use of content.

For more information about how to cite these materials visit http://open.umich.edu/education/about/terms-of-use.

Any medical information in this material is intended to inform and educate and is not a tool for self-diagnosis or a
replacement for medical evaluation, advice, diagnosis or treatment by a healthcare professional. Please speak to your
physician if you have questions about your medical condition.

Viewer discretion is advised: Some medical content is graphic and may not be suitable for all viewers.
Attribution Key
                   for more information see: http://open.umich.edu/wiki/AttributionPolicy



Use + Share + Adapt
  { Content the copyright holder, author, or law permits you to use, share and adapt. }
              Public Domain – Government: Works that are produced by the U.S. Government. (17 USC § 105)
              Public Domain – Expired: Works that are no longer protected due to an expired copyright term.
              Public Domain – Self Dedicated: Works that a copyright holder has dedicated to the public domain.

              Creative Commons – Zero Waiver

              Creative Commons – Attribution License
              Creative Commons – Attribution Share Alike License
              Creative Commons – Attribution Noncommercial License
              Creative Commons – Attribution Noncommercial Share Alike License
              GNU – Free Documentation License

Make Your Own Assessment
   { Content Open.Michigan believes can be used, shared, and adapted because it is ineligible for copyright. }
              Public Domain – Ineligible: Works that are ineligible for copyright protection in the U.S. (17 USC § 102(b)) *laws in
              your jurisdiction may differ
  { Content Open.Michigan has used under a Fair Use determination. }
              Fair Use: Use of works that is determined to be Fair consistent with the U.S. Copyright Act. (17 USC § 107) *laws in
              your jurisdiction may differ
              Our determination DOES NOT mean that all uses of this 3rd-party content are Fair Uses and we DO NOT guarantee
              that your use of the content is Fair.
              To use this content you should do your own independent analysis to determine whether or not your use will be Fair.
Problem Solving Cases
Learning Objectives
•   After attending one or more these eleven 30 minute sessions the student should be able
    to:
•
•   1. Demonstrate increased competence in using knowledge obtained from lectures and
    textbooks to answer patient questions about their diseases.
•
•   2. Demonstrate increased competence at correctly interpreting laboratory tests for liver
    disease, viral hepatitis, analysis of stools samples and analysis of ascites samples.
•
•   3. Demonstrate increased competence in identifying abnormalities on radiographic studies
    and suggesting a diagnosis (or diagnoses).
•
•   4. Demonstrate increased competence in selecting drug treatments for GERD, diarrhea.
•
•   5. Demonstrate increased competence in identifying what complications might occur when
    patients undergo certain GI surgical procedures and how these may be managed.
•
•   6. Demonstrate increased problem-solving skills for patients with GI diseases.
Industry Relationship
         Disclosures
 Industry Supported Research and
       Outside Relationships
• None
January 26, 2012
Your patient has an endoscopy and these pictures were
 obtained. What problems might this patient have or
      develop in the future? Why did this occur?
Case A

• A 24 year old medical student has
  developed epigastric pain.
• She thinks she has an ulcer.
• How would you determine whether she has
  an ulcer?
• Is there a “best” approach?
Case – B

 A 75 year old patient comes to see you as a new patient in
geriatrics clinic. In taking a history you discover he had surgery
for recurrent stomach ulcers in 1963. He thinks part of his
stomach was removed at the time.

What kind of operation did he likely have?

What is his current anatomy likely to be?

What problems can occur with these types of surgery?
Case C

• 75 year old woman is concerned she may
  have gastric cancer.
• This disease arises in the gastric epithelium.
• How would you try to find out if she has
  gastric cancer?
• What can you do?
• Is there a “best”way to answer her
  question?
Case - D
• The family is gathered for a Super Bowl party,
  complete with all the usual munchies.
• Your uncle pulls you aside and tells you he gets
  bad heartburn from the salsa, which he loves.
• Now that you are a medical student, he want
  advice on how to prevent the heartburn.
• What do you suggest?
Case – D1
• Three hours later your cousin, who gorged on
  pizza, complains of terrible heartburn and wants
  you to do something RIGHT NOW to relieve her
  symptoms.
• What do you suggest?
Case – D2
• Your grandmother overhears these conversations
  and loudly complains that her doctor has told her
  she is susceptible to stomach ulcers and therefore
  she cannot take her “arthritis” pills.
• She has severe pain in her hands, hip and knees
  and wants to know why the doctor took away her
  medications and what you can do to solve her
  problem.
• What pills were removed and why?
• What options are available?
Case E
• A 60 year old woman went to the Mayo Clinic
  and was told she had Zollinger-Ellison syndrome.
• She returns to Ann Arbor and comes to you for
  care.
• You recall the Z-E syndrome is due to a small
  gastrin-secreting tumor.
• What problems might you expect her to develop?
• What signs and symptoms might she develop?
• What could you do to help treat or prevent these
  problems?
Case F
• A patient comes to see you having been told he
  has a duodenal ulcer.
• He wants to know how/why he got the ulcer.
• What do you tell him (patient education)?
• What can he do to heal this ulcer as fast as
  possible (treatment)?
• What can he do to prevent future ulcers
  (secondary prevention)?
January 27, 2012
• A 29 year old man come to see you because
  of recurrent gas and diarrhea.
• He wants to know:
• Does he have lactose intolerance?
• Does milk cause his symptoms?
• Does he have lactase deficiency?
• How would you answer each question for
  him?
Case – G
A 55 year old man comes to your office for evaluation of diarrhea.
Diarrhea began in the past year although he cannot pinpoint an
exact time. He notes 3-5 loose stools during the day and none at
night.

He has no abdominal pain and has not lost weight.
His only other medical problem is frequent heartburn for which he
takes antacids.
The physical examination is normal except the digital rectal
examination which yields loose/watery light brown stool that is
negative for occult blood.
                What is your differential diagnosis?
                What do you do next?
Case – G-2
Diarrhea continues after he stops the antacid intake

Stool electrolytes:   Na = 90 mEq/l
                      K = 40
                      Cl = 40

Stool/plasma osmolality = 295

Stool osmotic gap = ????

Diagnostic possibilities?
Analysis of Fecal Electrolytes – normal values

 Sodium                   ~20-40 mEq/l
 Potassium                ~90
 Chloride                 ~15
 HCO3-                    ~30
 Anions (SO4-2,           ~85
    PO4-3, fatty acids)
 Magnesium                ~10-20

 Volume                   0.2-0.4 liters/day

 Plasma osmolality        ~290-300 mEq/l
 Stool osmolality         ~290-300 mEq/l
 Osmotic gap              ~40-70

 Fecal pH                 > ~5.4
Analysis of Fecal Electrolytes - I
  Sodium                 105
  Potassium               30
  Chloride                69
  Magnesium               15

  Electrolyte pattern?

  Volume                 3 liters/day

  Plasma osmolality      295 mEq/l
  Stool osmolality       301 mEq/l

  Osmotic gap?

  Fecal pH               > 5.4
Analysis of Fecal Electrolytes - II
   Sodium                 22
   Potassium              26
   Chloride               55


   Electrolyte pattern?

   Volume                 1.3 liters/day

   Plasma osmolality      295 mEq/l
   Stool osmolality       299 mEq/l

   Osmotic gap?

   Fecal pH               > 5.4
Analysis of Fecal Electrolytes - III
   Sodium                 43
   Potassium              89
   Chloride               18
   Magnesium              18

   Electrolyte pattern?

   Volume                 0.28 liters/day

   Plasma osmolality      295 mEq/l
   Stool osmolality       302 mEq/l

   Osmotic gap?
Analysis of Fecal Electrolytes - IV
   Sodium                 32
   Potassium              28
   Chloride               10


   Electrolyte pattern?

   Volume                 1.5 liters/day

   Plasma osmolality      295 mEq/l

   Osmotic gap?
Analysis of Fecal Electrolytes - V
   Sodium                 20
   Potassium              45
   Chloride               10
   Magnesium              10

   Electrolyte pattern?

   Volume                 1.1 liters/day

   Plasma osmolality      295 mEq/l
   Stool osmolality       140 mEq/l

   Osmotic gap?
Analysis of Fecal Electrolytes - VI
   Sodium                 103
   Potassium               42
   Chloride                18
   Magnesium               11

   Electrolyte pattern?

   Volume                 1.8 liters/day

   Plasma osmolality      295 mEq/l
   Stool osmolality       303 mEq/l

   Osmotic gap?
Analysis of Fecal Electrolytes – secretory
                diarrhea
       Sodium                   105
       Potassium                 30
       Chloride                  69
       Magnesium                 15

       Electrolyte pattern?
       Volume                    3 liters/day
       Plasma osmolality        295 mEq/l
       Stool osmolality         301 mEq/l
       Osmotic gap?
       Fecal pH                 > 5.4

       High sodium, no osmotic gap, normal pH,
       high volume = secretory diarrhea
January 30, 2012
Case – H I

 An 85 year old woman with untreated atrial
fibrillation throws an embolus that lodges in
the superior mesenteric artery proximal to
the origin of the ileocolic artery causing
ischemic necrosis of what part of the bowel?
Case – H II




              200 cm ileum
Case – H III

She undergoes emergency surgery with resection
of 200cm of this part of the bowel and anastamosis
of the proximal ileum to the transverse colon.


She recovers uneventfully,however what problems
might she develop due to loss of this bowel?
Case – I- I
• A patient has systemic sclerosis
  (scleroderma).

• You read in your textbook that this disease
  destroys GI smooth muscle, first in the
  esophagus, later small bowel and colon.

• The patient wants to know what problems
  she is likely to develop in the future.

• What do you tell her?
Case J
• A patient travels to South America and is bitten by
  the reduviid bug, transmitting Trypanosoma cruzi.

• He develops Chagas disease.

• You recall your parasitology and that this
  trypanosome specifically involves the wall of the
  esophagus and LES destroying NO-secreting
  neurons.

• What signs and symptoms will the patient develop
  and why?
Case K
• A patient comes to you requesting an
  injection of botulinus toxin in the lower
  esophageal sphincter.

• What will this toxin do in this location?

• What type of symptoms might be
  expected to be relieved?

• What complications might occur?
Case L

A 75 year old man comes to see you because of
diarrhea.

He has 5-7 loose watery stools a day and urgency.
He says the volume of the stools are moderate.

You evaluate him and find on biopsy that he has
microscopic colitis. You remember that this is a
chronic disorder that cannot be cured.

What treatment options are available?

What would you advise him to do?
Case M
A 28 year old woman comes to see you
because of urgency, diarrhea and bright red
blood mixed with some of her stools. This has
been going on for about 2 months.

You evaluate her and make a diagnosis of left-
sided ulcerative colitis.

What problems need management?

What types of treatment might you give her?
Case N
A 35 year old woman comes to you for evaluation of diarrhea
and weight loss. Last year she had surgery for active Crohn’s
disease at which time over a meter (>100 cm) of distal ileum
was resected.

She recovered well from the surgery but then developed
frequent soft stools and has lost about 20 pounds of weight.

What pathophysiologic explanation can you develop
for her diarrhea and how would you test your hypothesis?

What other problem(s) would you look for in this patient?
Case – O- I
• A patient comes to see you saying a doctor
  told him he had “colitis”.
• He has daily soft or liquid stools and thinks
  he was told he had ulcerative colitis, but is
  not sure.
• He occasionally sees streaks of blood.
• Symptoms have been present for 6 months.
• What diagnoses are possible?
• How could you determine the type of
  “colitis”?
Case – O- II
• The same patient returns with medical records
  including a colonoscopy report.

• The colonoscopy showed inflammation and ulcers
  located only in the sigmoid colon and in the
  cecum.

• What do you think the correct diagnosis is?
• How would you prove this?
• He continues to have symptoms - how would you
  treat him?
January 31, 2012
Case P
• A 30 year old business woman has worsening
  diarrhea, now 3-6 times a day, especially after
  meals. None at night
• She had these symptoms for several years but
  they have worsened lately as business is worse.
• Every few weeks she gets constipation for 3-4
  days, then diarrhea.
• She gets crampy abdominal pains on many days.
• What problems could cause her symptoms?
• How would you evaluate and treat her?
Case – Q- I
An 18 year old girl presents with an 8 week history of mild mid-
abdominal pain, diarrhea, and weight loss. The pain is
described as “achy” but not very specific. She notices it more at
night when she is trying to sleep.

She now has 3-5 soft semi-formed bowel movements per day
and occasionally she has to get up at night to pass stool.

She notes anorexia and “loss of energy” and has lost 5 pounds
in weight. She has not seen blood in her stools. She has not
traveled in the last year and knows no one with diarrhea. She
takes no medications.
              What is your differential diagnosis?
              What do you do next?
              Any lab tests you want?
Case – Q- II

WBC         10,000             (nl 4,000-8,000)
Hct         32                 (nl 36-45)
MCV         95                 (nl 80-99)
Platelets   250,000            (nl 150,000-350,000)

Electrolytes, BUN, Cr normal

What have you learned?

What can you think of to do next?
What do these images show?




What signs or symptoms might this patient have had?
Case R


• A patient is brought into the ER having
  vomited bright red blood 6 times this
  morning, each time “cups and cups” of
  blood.
• What do you do first?
• What are the diagnostic possibilities?
Case S

You have a patient in whom food does not pass
out of the stomach normally.

What symptoms do you expect?

What underlying diseases might the patient have?
February 1, 2012
Case T-1
       A 43 year old woman is brought to the emergency room
because of vomiting blood. Yesterday she felt somewhat weak,
nauseated and not her usual self. This morning she began vomiting,
brought up large amounts of bright red blood and clots and felt dizzy.
       She has been taking ibuprofen for the past three weeks
because of shoulder bursitis. She has a long history of taking
antacids for burning epigastric and substernal pain that occurs
3-5 times per week between meals or at night.
       Four years ago when she tried to donate blood, she was told
she could not do so because she had abnormal liver tests.
She does not smoke and no longer drinks alcohol although she
drank regularly until she was 35.
What is her main problem when you see her in the ER?
What do you want to do next?
Case T - 2
On physical examination she looks reasonably well.

       Lying: BP 110/60, P85         Sitting: BP 90/45, P110

Examination is normal. She has active bowel sounds,
but epigastric tenderness upon palpation without any masses.
Stool obtained by digital rectal examination is burgundy/black,
soft and markedly positive by hemoccult testing.

       How much has this woman bled?
       What else do you want to know?
Case T - 3
WBC             7.5        (nl ≤ 10)
Hct.            22         (nl 36 - 45)
Platelets       200,000    (nl 150,000 - 350,000)
PT              12 sec     (nl 9 - 12.5 sec)

What does this tell you?

What do you do next?
Case U

A 35 year old man comes to see you for a general medical
examination.

When you take a history, you find out that his paternal
grandfather, father’s brother and his older brother all have
colon cancer.

What are your concerns about your patient?

What more information might you want to get from the
history?

What do you advise your patient to do and why?
Case V
• A patient comes to the ER with severe and
  frequent nausea and vomiting of food and gastric
  contents after virtually every meal for the past 6
  weeks.
• She has lost 15 pounds
• On exam she is thin with orthostatic changes in BP
  and pulse.
• What electrolyte/blood test abnormalities might you
  expect?
• What types of problems could cause these
  symptoms?
• How do you prove what is causing her problem?
How do you educate and treat each
   of these patients? Cases W 1-3
• Your 11 year old future Olympic ice skater gets
  nauseated and often vomits after her fast spins.

• Your pregnant sister starts vomiting every
  morning.

• A patient with lung cancer tells you he cannot
  continue chemotherapy as his vomiting is too
  severe.
February 2, 2012
Case X
• A 45 year old woman comes to see you as
  she has started passing gas and stool
  through her vagina.
• Needless to say she is very distressed and
  wants this solved immediately.
• What had to have happened?
• How can you prove it?
• Later this week, think about what kind of
  diseases could have caused this problem.
What is abnormal on this x-ray and what
                is it?
What do you see on this x-ray?
What does this x-ray show?
What would you find on exam?
What do you see in this image – the abdominal x-ray?
CT enterography of normal abdomen for your interest
Case Y

• A patient has a “lupus anticoagulant” or
  anti-phospholipid antibody and develops a
  portal vein thrombosis with complete
  obstruction of portal venous blood flow.

• What complications would you expect to
  occur?
Analysis of Liver Tests
Case 1
                 Laboratory Findings

Bilirubin              8.5             (0.2-1.2 mg/dl)

Alkaline Phos.         250             (23-100 IU/ml)

AST                    1500            (20-35 IU/ml)

ALT                    1750            (18-30 IU/ml)

Albumin                4.0             (3.5-4.5 g/dl)

PT                     11.0            (10.5-12.0 sec)
Case 2
                 Laboratory Findings

Bilirubin              8.5             (0.2-1.2 mg/dl)

Alkaline Phos.         675             (23-100 IU/ml)

AST                    92              (20-35 IU/ml)

ALT                    99              (18-30 IU/ml)

Albumin                4.0             (3.5-4.5 g/dl)

PT                     11.5            (10.5-12.0 sec)
Case 3
                 Laboratory Findings

Bilirubin              3.5             (0.2-1.2 mg/dl)

Alkaline Phos.         190             (23-100 IU/ml)

AST                    300             (20-35 IU/ml)

ALT                    400             (18-30 IU/ml)

Albumin                4.0             (3.5-4.5 g/dl)

PT                     12.0            (10.5-12.0 sec)
Case 4
                 Laboratory Findings

Bilirubin              0.8             (0.2-1.2 mg/dl)

Alkaline Phos.         90              (23-100 IU/ml)

AST                    2500            (20-35 IU/ml)

ALT                    28              (18-30 IU/ml)

Albumin                4.0             (3.5-4.5 g/dl)

PT                     11.0            (10.5-12.0 sec)
Case 5
                 Laboratory Findings

Bilirubin              9.0             (0.2-1.2 mg/dl)

Alkaline Phos.         175             (23-100 IU/ml)

AST                    210             (20-35 IU/ml)

ALT                    100             (18-30 IU/ml)

Albumin                3.2             (3.5-4.5 g/dl)

PT                     14.5            (10.5-12.0 sec)
Case 6
                 Laboratory Findings

Bilirubin              9.0             (0.2-1.2 mg/dl)

Alkaline Phos.         200             (23-100 IU/ml)

AST                    2500            (20-35 IU/ml)

ALT                    3200            (18-30 IU/ml)

Albumin                4.0             (3.5-4.5 g/dl)

PT                     14.5            (10.5-12.0 sec)
Case 7
                 Laboratory Findings

Bilirubin              1.0             (0.2-1.2 mg/dl)

Alkaline Phos.         555             (23-100 IU/ml)

AST                    20              (20-35 IU/ml)

ALT                    22              (18-30 IU/ml)

Albumin                4.0             (3.5-4.5 g/dl)

PT                     11.5            (10.5-12.0 sec)
Case 8
                 Laboratory Findings

Bilirubin              3.0             (0.2-1.2 mg/dl)

Alkaline Phos.         120             (23-100 IU/ml)

AST                    65              (20-35 IU/ml)

ALT                    68              (18-30 IU/ml)

Albumin                2.0             (3.5-4.5 g/dl)

PT                     15.5            (10.5-12.0 sec)
February 3, 2012
Analysis of Serologic Tests for Viral
            Hepatitis – Case Z
• A 23 year old medical student comes to the
  emergency room with the following symptoms:
   – 1 week of nausea, vomiting, severe fatigue and 1 day of
     jaundice
• Lab tests are:
   –   bilirubin:             5.6             (nl <1.1 mg/dl)
   –   AST/ALT                1500/1900       (nl<70 IU)
   –   Alk Phos               330             (nl<110 IU)
   –   Prothrombin time       11.5 sec                 (nl<12
       sec)
• You send every serologic test you can think of – the
  results come back and you have to interpret them.
Analysis of Hepatitis Tests

Hepatitis A   positive   IgM antibody
              negative   IgG antibody

Hepatitis B   negative   sAg (surface antigen)
              positive   sAB (surface antibody)
              negative   IgM antibody to core
              negative   IgG antibody to core
              negative   eAg (e antigen)
              negative   eAB (e antibody)
              negative   DNA

Hepatitis C   negative   antibody (IgG)
              negative   RNA by PCR

Hepatitis D   negative   RNA by PCR
              negative   IgM antibody
              negative   IgG antibody

Hepatitis E   negative   IgM antibody
              negative   IgG antibody
Analysis of Hepatitis Tests

Hepatitis A   negative   IgM antibody
              negative   IgG antibody

Hepatitis B   negative   sAg (surface antigen)
              positive   sAB (surface antibody)
              negative   IgM antibody to core
              positive   IgG antibody to core
              negative   eAg (e antigen)
              negative   eAB (e antibody)
              negative   DNA

Hepatitis C   negative   antibody (IgG)
              positive   RNA by PCR

Hepatitis D   negative   RNA by PCR
              negative   IgM antibody
              negative   IgG antibody

Hepatitis E   negative   IgM antibody
              negative   IgG antibody
Analysis of Hepatitis Tests

Hepatitis A   negative   IgM antibody
              positive   IgG antibody

Hepatitis B   negative   sAg (surface antigen)
              negative   IgM antibody to core
              negative   IgG antibody to core
              negative   eAg (e antigen)
              negative   eAB (e antibody)
              negative   DNA

Hepatitis C   negative   antibody (IgG)
              negative   RNA by PCR

Hepatitis D   negative   RNA by PCR
              negative   IgM antibody
              negative   IgG antibody

Hepatitis E   positive   IgM antibody
              negative   IgG antibody
Analysis of Hepatitis Tests

Hepatitis A   negative   IgM antibody
              positive   IgG antibody

Hepatitis B   positive   sAg (surface antigen)
              negative   sAB (surface antibody)
              positive   IgM antibody to core
              negative   IgG antibody to core
              positive   eAg (e antigen)
              negative   eAB (e antibody)
              positive   DNA

Hepatitis C   negative   antibody (IgG)
              negative   RNA by PCR

Hepatitis D   negative   RNA by PCR
              negative   IgM antibody
              negative   IgG antibody

Hepatitis E   negative   IgM antibody
              negative   IgG antibody
Analysis of Hepatitis Tests

Hepatitis A   negative   IgM antibody
              positive   IgG antibody

Hepatitis B   positive   sAg (surface antigen)
              negative   sAB (surface antibody)
              negative   IgM antibody to core
              positive   IgG antibody to core
              negative   eAg (e antigen)
              positive   eAB (e antibody)
              negative   DNA

Hepatitis C   negative   antibody (IgG)
              negative   RNA by PCR

Hepatitis D   positive   RNA by PCR
              positive   IgM antibody
              negative   IgG antibody

Hepatitis E   negative   IgM antibody
              negative   IgG antibody
Analysis of Hepatitis Tests

Hepatitis A   negative   IgM antibody
              negative   IgG antibody

Hepatitis B   negative   sAg (surface antigen)
              negative   sAB (surface antibody)
              negative   IgM antibody to core
              negative   IgG antibody to core
              negative   eAg (e antigen)
              negative   eAB (e antibody)
              negative   DNA

Hepatitis C   positive   antibody (IgG)
              positive   RNA by PCR

Hepatitis D   negative   RNA by PCR
              negative   IgM antibody
              negative   IgG antibody

Hepatitis E   positive   IgM antibody
              negative   IgG antibody
Case AA
• A 32 year old man had developed a greatly
  enlarged abdomen over the past several
  months.

• He asks you what this is due to.

• What are the possible causes of his enlarged
  abdomen?

• What can you do to investigate the cause of
  this enlargement?
A patient brings you a liver biopsy. Here
          is one image from it.
 What does the patient have and what
  problems might the patient develop?
Case BB - A patient comes to see you with liver biopsy
slides. This is what you see. What is the problem and
      what symptoms might the patient develop?




     H&E stain
                                    Trichrome stain
February 6, 2012
Case CC
• A patient with cirrhosis wants to know if
  she has the following. How would you
  try to find out?
  – Hepatocellular carcinoma
  – Ascites
  – Esophageal varices
  – Hepatic hydrothorax
     • pleural effusion due to ascites
Case DD
• A 35 year old woman had a CT scan of
  the abdomen that showed a thrombus
  occluding her splenic vein.

• She wants to know what problems she
  will develop because of this.

• Why did she develop this thrombus?
What are these skin findings?
Case EE
• A patient with cirrhosis and portal
  hypertension is concerned about his
  health and reads advertisements for
  Gator-aid - that if makes you stronger
  and better able to exercise. He buys a
  large case and drinks 3-4 bottles per
  day.

• What problem(s) is he likely to develop?
Case FF
• A patient with cirrhosis and portal
  hypertension is concerned that his arms
  and legs are getting thinner.
• He starts to work out at a gym and
  consults the nutritionist there who
  advises a high protein diet with meat
  and protein powder supplements.

• What problem might he develop?
What do you see here?
What diagnoses are you thinking of?
Case GG
A patient is admitted to your service for evaluation of
      new onset ascites.

You perform a paracentesis and send the fluid for a variety
     of tests.

The tests come back and your resident asks you to interpret
      the results.
• Remember Dr. Moseley’s lecture on
  complications of cirrhosis and ascites

• When a patient presents with new onset
  ascites, diagnostic studies need to be done
  and analyzed to determine the cause of
  ascites and whether there is a complication
  (infection)
Ascites Fluid Analysis
Test                 Ascites             Serum (normal)

Total protein        2.2                 7.5    (8-9g/dl)

Albumin              1.0                 2.9    (3.5-5.0g/dl)

Cell count (PMNs)    10                  4000   (2000-6000)

Triglycerides        15                  150    (<160mg/dl)

Amylase              20                  40     (0-130 U/l)



How would you interpret these results?
- Peritonitis
Non-cirrhotic portal hypertension

“Hydrodynamic/transudative”         “Exudative”
Ascites Fluid Analysis
Test                    Ascites               Serum

Total protein           6.5                   7.5     (8-9g/dl)

Albumin                 3.5                   4.3        (3.5-5.0g/dl)

Cell count (PMNs)       10                    4000    (2000-6000)

Triglycerides           15                    150     (<160mg/dl)

Amylase                 20                    40      (0-130 U/l)


                How would you interpret these results?
                     What more could you do?
Ascites Fluid Analysis
Test                 Ascites             Serum

Total protein        7.4                 7.5     (8-9g/dl)

Albumin              3.4                 4.1     (3.5-5.0g/dl)

Cell count (PMNs)    10                  4000    (2000-6000)

Triglycerides        15                  150     (<160mg/dl)

Amylase              950                 40      (0-130 U/l)



How would you interpret these results?
Ascites Fluid Analysis
Test                 Ascites             Serum

Total protein        6.5                 7.5     (8-9g/dl)

Albumin              3.6                 4.5     (3.5-5.0g/dl)

Cell count (PMNs)    10                  4000    (2000-6000)

Triglycerides        820                 150     (<160mg/dl)

Amylase              20                  40      (0-130 U/l)



How would you interpret these results?
Ascites Fluid Analysis
Test                 Ascites             Serum (normal)

Total protein        3.5                 7.5    (8-9g/dl)

Albumin              2.5                 4.5    (3.5-5.0g/dl)

Cell count (PMNs)    10                  4000   (2000-6000)

Triglycerides        15                  150    (<160mg/dl)

Amylase              20                  40     (0-130 U/l)



How would you interpret these results?
Ascites Fluid Analysis
Test                 Ascites             Serum (normal)

Total protein        2.6                 7.6    (8-9g/dl)

Albumin              1.0                 2.8    (3.5-5.0g/dl)

Cell count (PMNs)    774                 8300   (2000-6000)

Triglycerides        15                  150    (<160mg/dl)

Amylase              20                  40     (0-130 U/l)



How would you interpret these results?
Ascites Fluid Analysis
Test                 Ascites             Serum (normal)

Total protein        6.8                 7.6   (8-9g/dl)

Albumin              3.9                 4.3    (3.5-5.0g/dl)

Cell count (PMNs)    10,200              18,950 (2000-6000)

Triglycerides        15                  150   (<160mg/dl)

Amylase              20                  40    (0-130 U/l)

How would you interpret these results?
What other tests might be abnormal?
Ascites Fluid Analysis
Test                Ascites   Serum (normal)

Total protein       6.8       7.6   (8-9 g/dl)

Albumin             3.9       4.3    (3.5-5.0 g/dl)

Cell count (PMNs)   10,200    18,950 (2000-6000)

Triglycerides       15        142   (<150 mg/dl)

Amylase             20        40    (0-130 U/l)

LDH                 495       150   (120-240 IU/l)

Glucose             15        112   (73-110 mg/dl)
February 7, 2012
Acute hepatitis B case from small
        group yesterday

• What is this patient’s prognosis (what
  may happen to him)?

• He has an infectious disease – what do
  you need to do when wearing your
  public health hat?
Case HH
• A patient of yours takes a drug (A) that is
  metabolized in the liver primarily by
  CYP3A4
• She develops a new problem and you give
  her another drug (B). However it is also
  metabolized in the liver by CYP3A4
• What problems could occur?
Case II
• You are treating a patient for tuberculosis
  with isoniazid. You read that this is
  primarily metabolized by CYP2E1.
• Your patient tells you he drinks alcohol
  regularily - up to 6 beers per day plus some
  liquor on the weekends.
• Does this concern you?
Case JJ
 A 35 year old man presents with 18 hours of pain that began in
the periumbilical region as an “aching” pain. Subsequently,
the pain became more severe and was felt also in the right lower
quadrant. He has vomited stomach contents several times and feels
nauseated.
 Examination shows a pale sweaty man curled on his side in pain.
Pulse is 110 and he is febrile (T 100.8˚F). Bowel sounds are absent.
He is very tender in the right lower quadrant and “guards” in this
area.
When the examiner pushes down on the left lower quadrant and
releases quickly, severe pain is felt in the right lower quadrant.

 Laboratory studies are essentially normal except for a leukocytosis
(WBC 13,000).

              What is your differential diagnosis?
Case KK - 1: This patient has epigastric pain
 and nausea/vomiting. What do you see?
Case LL

• A patient develops localized carcinoma
  of the ampulla of Vater.
• This is treated by surgical resection of
  the duodenum and part of the
  pancreas. The stomach and
  biliary/pancreatic ducts are each
  anastamosed to loops of jejunum.
• What problems might develop?
Patient Case MM
• 65 year old comes in with left lower
  quadrant abdominal aching pain for the last
  2 days with fever
• He is concerned he may have acute
  diverticulitis as he has had similar
  symptoms in the past.
• How would you determine whether he had
  diverticulitis?
February 8, 2012
Case NN
• A patient has an abdominal x-ray performed
  for an unrelated reason.
• The radiologist reports the presence of
  calcifications throughout the pancreas
• The patient wants to know what this means?
• What do you tell her?
• What might you want to do with her?
Case OO
• You are caring for a patient with acute
  pancreatitis
• The patient asks you what complications he
  might develop – what do you tell him?
• He wants to know how he got this disease –
  what do you tell him?
Case PP-1
• You are taking care of a patient with acute
  pancreatitis and the resident asks you how
  sick the patient is.
• What factors can you assess to tell if this
  patient is severely ill with pancreatitis or
  not?
Case QQ
• A 20 year old girl was involved in a severe
  MVA. She was driving and had a crush injury
  to her pancreas from the steering wheel.

• She develops severe acute pancreatitis and
  requires surgery to resect the necrotic tissue
  and she loses about 80% of her pancreas.

• She survives but is likely to develop what
  complications?
Case RR

Three patients are admitted to your service-
1. one has nausea, vomiting and dehydration
   due to gastric outlet obstruction,
2. one is jaundiced, and
3. one has severe epigastric pain and weight loss.

What single disease could they all have?
Case SS
• A patient comes to see you with 6 months of
  chronic abdominal pain thought to be due to
  chronic pancreatitis (from alcohol).
• Upon evaluation you determine the patient has lost
  about 35 pounds of weight over this period and
  has malabsorption due to pancreatic insufficiency.
• Why did the patient develop pancreatic
  insufficiency?
• How can you treat the pain?
• How can you treat the pancreatic insufficiency?
Case TT - 1
A 35 year old man is brought to the emergency room with a one
day history of abdominal pain. The patient felt well until 18
hours ago when he developed constant epigastric pain that he
describes as “boring through” to the back associated with
nausea and vomiting of gastric contents. He feels somewhat better
if he sits up bent forward.

On examination he is pale, sweaty and in pain. Vital signs are
normal except a pulse of 100. Abdominal exam shows hypoactive
infrequent bowel sounds and severe epigastric pain on even mild
palpation. There is no “rebound” tenderness.

              What is your differential diagnosis?
              What do you do next?
Case TT - 2
WBC            14,000        (nl ≤ 10)
Hct            45            (nl 36 - 45)
Platelets      250,000       (nl 150,000 - 350,000)
Elecrolytes, Bun, Cr normal
AST                   145 I.U.              (nl < 45)
ALT                   160 I.U.              (nl < 50)
Alk phos              290 I.U.              (nl < 110)
Bili                  2.0 mg/dl             (nl < 1.0)
Albumin               4.5 gm/dl             (nl 3.5-4.5)
PT                    10.5 sec              (nl 10-12.5)
Amylase               10 X normal
Lipase                20 X normal

Abdominal x-ray shows no free air
Case UU
• A patient with pancreatic cancer develops
  jaundice.
• Why did the jaundice develop?
• What complications will occur?
• What can be done about the jaundice?
February 9, 2012
What do you see in this x-ray?
What do you see here?
  What diseases could cause this?
What accompanying problems might you
             expect?
Case UU
A patient comes to see you as he had an x-ray done
when he went to the Emergency Room for abdominal
pain and it showed gallstones.

He wants to know if he should have his gallbladder
taken out.

Who should have their gallbladder removed?

What signs and symptoms might he have had that
would convince you he should have a
cholecystectomy?
VV - Each of the following patients
  has gallstones – what type?
• 45 year old woman who is obese and has had 4
  children.

• A 21 year old man with sickle cell disease.

• A 25 year old woman with Crohn’s disease who has
  had >100 cm of her terminal ileum resected.

• A 50 year old who lost most of his small bowel due to
  a gunshot wound and receives nutrition through a
  central vein (total parenteral nutrition).
What do you see here?
75 year old man with diabetes and history of acute right upper
     quadrant pain, fever, elevated white blood count. This
               is emphysematous cholecystitis.
    Cultures of the gallbladder grew Clostridium perfringens



 Air in GB,
 air/fluid level

Air in the
wall of the GB
Case VV
A 35 year old man presents with 12 hours of pain that began as a
constant “aching”, “sharp” mid abdominal pain that slowly moved
to the right upper quadrant.

The patient is tachycardic (P=98) with a low grade fever (100˚F)
 and on abdominal examination has moderate pain on palpation
in the periumbilical/epigastric region and severe pain in the right
upper quadrant.

Laboratory studies are normal except for a mild leukocytosis
(WBC 11,000).
              What is your differential diagnosis?
              What do you do next?
Case WW
• A patient has pain in the epigastric region that began
  2 days ago. What organs could be reponsible for this
  pain?

• What do each of the following pain patterns suggest
  as etiologies?

   – Sharp/severe penetrating pain radiating through to
     the back.
   – Dull aching pain that increased over several hours
     and then moved to the RUQ?
   – Burning epigastric pain that did not radiate?

More Related Content

Similar to Problem Solving Sessions

10.01.08: Potassium and Magnesium Homeostasis
10.01.08: Potassium and Magnesium Homeostasis10.01.08: Potassium and Magnesium Homeostasis
10.01.08: Potassium and Magnesium HomeostasisOpen.Michigan
 
09.26.08: Course Introduction
09.26.08: Course Introduction09.26.08: Course Introduction
09.26.08: Course IntroductionOpen.Michigan
 
GEMC- Test-Taking Skills- Resident Training
GEMC- Test-Taking Skills- Resident TrainingGEMC- Test-Taking Skills- Resident Training
GEMC- Test-Taking Skills- Resident TrainingOpen.Michigan
 
GEMC - Abdominal Emergencies- For Nurses
GEMC - Abdominal Emergencies- For NursesGEMC - Abdominal Emergencies- For Nurses
GEMC - Abdominal Emergencies- For NursesOpen.Michigan
 
GEMC: Sepsis in the ED: Resident Training
GEMC: Sepsis in the ED: Resident TrainingGEMC: Sepsis in the ED: Resident Training
GEMC: Sepsis in the ED: Resident TrainingOpen.Michigan
 
GEMC: Procedural Sedation in the Emergency Department: Resident Training
GEMC: Procedural Sedation in the Emergency Department: Resident TrainingGEMC: Procedural Sedation in the Emergency Department: Resident Training
GEMC: Procedural Sedation in the Emergency Department: Resident TrainingOpen.Michigan
 
GEMC- Adrenal Insufficiency Crisis- for Residents
GEMC- Adrenal Insufficiency Crisis- for ResidentsGEMC- Adrenal Insufficiency Crisis- for Residents
GEMC- Adrenal Insufficiency Crisis- for ResidentsOpen.Michigan
 
GEMC: Non-Traumatic Abdominal Pain/Abdominal Emergencies: Resident Training
GEMC: Non-Traumatic Abdominal Pain/Abdominal Emergencies: Resident TrainingGEMC: Non-Traumatic Abdominal Pain/Abdominal Emergencies: Resident Training
GEMC: Non-Traumatic Abdominal Pain/Abdominal Emergencies: Resident TrainingOpen.Michigan
 
GEMC - Non-Traumatic Abdominal Pain/Abdominal Emergencies- Resident Training
GEMC - Non-Traumatic Abdominal Pain/Abdominal Emergencies- Resident TrainingGEMC - Non-Traumatic Abdominal Pain/Abdominal Emergencies- Resident Training
GEMC - Non-Traumatic Abdominal Pain/Abdominal Emergencies- Resident TrainingOpen.Michigan
 
03.03.09: Calcium Metabolism
03.03.09: Calcium Metabolism03.03.09: Calcium Metabolism
03.03.09: Calcium MetabolismOpen.Michigan
 
GEMC: Toxic Alcohols: Resident Training
GEMC: Toxic Alcohols: Resident TrainingGEMC: Toxic Alcohols: Resident Training
GEMC: Toxic Alcohols: Resident TrainingOpen.Michigan
 
GMEC - Fluid and Electrolyte Imbalances in Emergency Nursing
GMEC - Fluid and Electrolyte Imbalances in Emergency NursingGMEC - Fluid and Electrolyte Imbalances in Emergency Nursing
GMEC - Fluid and Electrolyte Imbalances in Emergency NursingOpen.Michigan
 
Kub guide without tos
Kub guide without tosKub guide without tos
Kub guide without tosMuhammad Saim
 
01.25.12: Introduction to M2 GI Sequence
01.25.12: Introduction to M2 GI Sequence01.25.12: Introduction to M2 GI Sequence
01.25.12: Introduction to M2 GI SequenceOpen.Michigan
 
Diagnostic Reasoning I and II
Diagnostic Reasoning I and IIDiagnostic Reasoning I and II
Diagnostic Reasoning I and IIOpen.Michigan
 
GEMC- Burns: Managements and Survivability- for Residents
GEMC- Burns: Managements and Survivability- for ResidentsGEMC- Burns: Managements and Survivability- for Residents
GEMC- Burns: Managements and Survivability- for ResidentsOpen.Michigan
 
GEMC: When Kidneys Fail
GEMC: When Kidneys FailGEMC: When Kidneys Fail
GEMC: When Kidneys FailOpen.Michigan
 
09.30.08(b): Approach to the Patient with Disorders of Osmoregulation
09.30.08(b): Approach to the Patient with Disorders of Osmoregulation09.30.08(b): Approach to the Patient with Disorders of Osmoregulation
09.30.08(b): Approach to the Patient with Disorders of OsmoregulationOpen.Michigan
 
CHEMISTRY IN EVERYDAY LIFE
CHEMISTRY IN EVERYDAY LIFECHEMISTRY IN EVERYDAY LIFE
CHEMISTRY IN EVERYDAY LIFEAJAL A J
 
The Value of Uncertainty
The Value of UncertaintyThe Value of Uncertainty
The Value of UncertaintyOpen.Michigan
 

Similar to Problem Solving Sessions (20)

10.01.08: Potassium and Magnesium Homeostasis
10.01.08: Potassium and Magnesium Homeostasis10.01.08: Potassium and Magnesium Homeostasis
10.01.08: Potassium and Magnesium Homeostasis
 
09.26.08: Course Introduction
09.26.08: Course Introduction09.26.08: Course Introduction
09.26.08: Course Introduction
 
GEMC- Test-Taking Skills- Resident Training
GEMC- Test-Taking Skills- Resident TrainingGEMC- Test-Taking Skills- Resident Training
GEMC- Test-Taking Skills- Resident Training
 
GEMC - Abdominal Emergencies- For Nurses
GEMC - Abdominal Emergencies- For NursesGEMC - Abdominal Emergencies- For Nurses
GEMC - Abdominal Emergencies- For Nurses
 
GEMC: Sepsis in the ED: Resident Training
GEMC: Sepsis in the ED: Resident TrainingGEMC: Sepsis in the ED: Resident Training
GEMC: Sepsis in the ED: Resident Training
 
GEMC: Procedural Sedation in the Emergency Department: Resident Training
GEMC: Procedural Sedation in the Emergency Department: Resident TrainingGEMC: Procedural Sedation in the Emergency Department: Resident Training
GEMC: Procedural Sedation in the Emergency Department: Resident Training
 
GEMC- Adrenal Insufficiency Crisis- for Residents
GEMC- Adrenal Insufficiency Crisis- for ResidentsGEMC- Adrenal Insufficiency Crisis- for Residents
GEMC- Adrenal Insufficiency Crisis- for Residents
 
GEMC: Non-Traumatic Abdominal Pain/Abdominal Emergencies: Resident Training
GEMC: Non-Traumatic Abdominal Pain/Abdominal Emergencies: Resident TrainingGEMC: Non-Traumatic Abdominal Pain/Abdominal Emergencies: Resident Training
GEMC: Non-Traumatic Abdominal Pain/Abdominal Emergencies: Resident Training
 
GEMC - Non-Traumatic Abdominal Pain/Abdominal Emergencies- Resident Training
GEMC - Non-Traumatic Abdominal Pain/Abdominal Emergencies- Resident TrainingGEMC - Non-Traumatic Abdominal Pain/Abdominal Emergencies- Resident Training
GEMC - Non-Traumatic Abdominal Pain/Abdominal Emergencies- Resident Training
 
03.03.09: Calcium Metabolism
03.03.09: Calcium Metabolism03.03.09: Calcium Metabolism
03.03.09: Calcium Metabolism
 
GEMC: Toxic Alcohols: Resident Training
GEMC: Toxic Alcohols: Resident TrainingGEMC: Toxic Alcohols: Resident Training
GEMC: Toxic Alcohols: Resident Training
 
GMEC - Fluid and Electrolyte Imbalances in Emergency Nursing
GMEC - Fluid and Electrolyte Imbalances in Emergency NursingGMEC - Fluid and Electrolyte Imbalances in Emergency Nursing
GMEC - Fluid and Electrolyte Imbalances in Emergency Nursing
 
Kub guide without tos
Kub guide without tosKub guide without tos
Kub guide without tos
 
01.25.12: Introduction to M2 GI Sequence
01.25.12: Introduction to M2 GI Sequence01.25.12: Introduction to M2 GI Sequence
01.25.12: Introduction to M2 GI Sequence
 
Diagnostic Reasoning I and II
Diagnostic Reasoning I and IIDiagnostic Reasoning I and II
Diagnostic Reasoning I and II
 
GEMC- Burns: Managements and Survivability- for Residents
GEMC- Burns: Managements and Survivability- for ResidentsGEMC- Burns: Managements and Survivability- for Residents
GEMC- Burns: Managements and Survivability- for Residents
 
GEMC: When Kidneys Fail
GEMC: When Kidneys FailGEMC: When Kidneys Fail
GEMC: When Kidneys Fail
 
09.30.08(b): Approach to the Patient with Disorders of Osmoregulation
09.30.08(b): Approach to the Patient with Disorders of Osmoregulation09.30.08(b): Approach to the Patient with Disorders of Osmoregulation
09.30.08(b): Approach to the Patient with Disorders of Osmoregulation
 
CHEMISTRY IN EVERYDAY LIFE
CHEMISTRY IN EVERYDAY LIFECHEMISTRY IN EVERYDAY LIFE
CHEMISTRY IN EVERYDAY LIFE
 
The Value of Uncertainty
The Value of UncertaintyThe Value of Uncertainty
The Value of Uncertainty
 

More from Open.Michigan

GEMC- Oncologic Emergencies- Resident Training
GEMC- Oncologic Emergencies- Resident TrainingGEMC- Oncologic Emergencies- Resident Training
GEMC- Oncologic Emergencies- Resident TrainingOpen.Michigan
 
GEMC- Cardiac Evalutation- Resident Training
GEMC- Cardiac Evalutation- Resident TrainingGEMC- Cardiac Evalutation- Resident Training
GEMC- Cardiac Evalutation- Resident TrainingOpen.Michigan
 
GEMC- Alterations in Body Temperature: The Adult Patient with a Fever- Reside...
GEMC- Alterations in Body Temperature: The Adult Patient with a Fever- Reside...GEMC- Alterations in Body Temperature: The Adult Patient with a Fever- Reside...
GEMC- Alterations in Body Temperature: The Adult Patient with a Fever- Reside...Open.Michigan
 
GEMC- Rapid Sequence Intubation & Emergency Airway Support in the Pediatric E...
GEMC- Rapid Sequence Intubation & Emergency Airway Support in the Pediatric E...GEMC- Rapid Sequence Intubation & Emergency Airway Support in the Pediatric E...
GEMC- Rapid Sequence Intubation & Emergency Airway Support in the Pediatric E...Open.Michigan
 
GEMC- Ocular Emgercencies- Resident Training
GEMC- Ocular Emgercencies- Resident TrainingGEMC- Ocular Emgercencies- Resident Training
GEMC- Ocular Emgercencies- Resident TrainingOpen.Michigan
 
GEMC- Disorders of the Pleura, Mediastinum, and Chest Wall- Resident Training
GEMC- Disorders of the Pleura, Mediastinum, and Chest Wall- Resident TrainingGEMC- Disorders of the Pleura, Mediastinum, and Chest Wall- Resident Training
GEMC- Disorders of the Pleura, Mediastinum, and Chest Wall- Resident TrainingOpen.Michigan
 
GEMC- Dental Emergencies and Common Dental Blocks- Resident Training
GEMC- Dental Emergencies and Common Dental Blocks- Resident TrainingGEMC- Dental Emergencies and Common Dental Blocks- Resident Training
GEMC- Dental Emergencies and Common Dental Blocks- Resident TrainingOpen.Michigan
 
GEMC- EMedHome Board Review: Procedures- Resident Training
GEMC- EMedHome Board Review: Procedures- Resident TrainingGEMC- EMedHome Board Review: Procedures- Resident Training
GEMC- EMedHome Board Review: Procedures- Resident TrainingOpen.Michigan
 
GEMC- Arthritis and Arthrocentesis- Resident Training
GEMC- Arthritis and Arthrocentesis- Resident TrainingGEMC- Arthritis and Arthrocentesis- Resident Training
GEMC- Arthritis and Arthrocentesis- Resident TrainingOpen.Michigan
 
GEMC- Bursitis, Tendonitis, Fibromyalgia, and RSD- Resident Training
GEMC- Bursitis, Tendonitis, Fibromyalgia, and RSD- Resident TrainingGEMC- Bursitis, Tendonitis, Fibromyalgia, and RSD- Resident Training
GEMC- Bursitis, Tendonitis, Fibromyalgia, and RSD- Resident TrainingOpen.Michigan
 
GEMC- Right Upper Quadrant Ultrasound- Resident Training
GEMC- Right Upper Quadrant Ultrasound- Resident TrainingGEMC- Right Upper Quadrant Ultrasound- Resident Training
GEMC- Right Upper Quadrant Ultrasound- Resident TrainingOpen.Michigan
 
GEMC- Cardiovascular Board Review Session 3- Resident Training
GEMC- Cardiovascular Board Review Session 3- Resident TrainingGEMC- Cardiovascular Board Review Session 3- Resident Training
GEMC- Cardiovascular Board Review Session 3- Resident TrainingOpen.Michigan
 
GEMC- Cardiovascular Board Review Session 2- Resident Training
GEMC- Cardiovascular Board Review Session 2- Resident TrainingGEMC- Cardiovascular Board Review Session 2- Resident Training
GEMC- Cardiovascular Board Review Session 2- Resident TrainingOpen.Michigan
 
GEMC- Cardiovascular Board Review Session 1- Resident Training
GEMC- Cardiovascular Board Review Session 1- Resident TrainingGEMC- Cardiovascular Board Review Session 1- Resident Training
GEMC- Cardiovascular Board Review Session 1- Resident TrainingOpen.Michigan
 
GEMC: Nursing Process and Linkage between Theory and Practice
GEMC: Nursing Process and Linkage between Theory and PracticeGEMC: Nursing Process and Linkage between Theory and Practice
GEMC: Nursing Process and Linkage between Theory and PracticeOpen.Michigan
 
2014 gemc-nursing-lapham-general survey and patient care management
2014 gemc-nursing-lapham-general survey and patient care management2014 gemc-nursing-lapham-general survey and patient care management
2014 gemc-nursing-lapham-general survey and patient care managementOpen.Michigan
 
GEMC: The Role of Radiography in the Initial Evaluation of C-Spine Trauma
GEMC: The Role of Radiography in the Initial Evaluation of C-Spine TraumaGEMC: The Role of Radiography in the Initial Evaluation of C-Spine Trauma
GEMC: The Role of Radiography in the Initial Evaluation of C-Spine TraumaOpen.Michigan
 
GEMC - Mammal and Human Bite Injuries
GEMC - Mammal and Human Bite InjuriesGEMC - Mammal and Human Bite Injuries
GEMC - Mammal and Human Bite InjuriesOpen.Michigan
 
GEMC- Sickle Cell Disease: Special Considerations in Pediatrics- Resident Tra...
GEMC- Sickle Cell Disease: Special Considerations in Pediatrics- Resident Tra...GEMC- Sickle Cell Disease: Special Considerations in Pediatrics- Resident Tra...
GEMC- Sickle Cell Disease: Special Considerations in Pediatrics- Resident Tra...Open.Michigan
 
GEMC- Ghana Grab Bag Pediatric Quiz- Resident Training
GEMC- Ghana Grab Bag Pediatric Quiz- Resident TrainingGEMC- Ghana Grab Bag Pediatric Quiz- Resident Training
GEMC- Ghana Grab Bag Pediatric Quiz- Resident TrainingOpen.Michigan
 

More from Open.Michigan (20)

GEMC- Oncologic Emergencies- Resident Training
GEMC- Oncologic Emergencies- Resident TrainingGEMC- Oncologic Emergencies- Resident Training
GEMC- Oncologic Emergencies- Resident Training
 
GEMC- Cardiac Evalutation- Resident Training
GEMC- Cardiac Evalutation- Resident TrainingGEMC- Cardiac Evalutation- Resident Training
GEMC- Cardiac Evalutation- Resident Training
 
GEMC- Alterations in Body Temperature: The Adult Patient with a Fever- Reside...
GEMC- Alterations in Body Temperature: The Adult Patient with a Fever- Reside...GEMC- Alterations in Body Temperature: The Adult Patient with a Fever- Reside...
GEMC- Alterations in Body Temperature: The Adult Patient with a Fever- Reside...
 
GEMC- Rapid Sequence Intubation & Emergency Airway Support in the Pediatric E...
GEMC- Rapid Sequence Intubation & Emergency Airway Support in the Pediatric E...GEMC- Rapid Sequence Intubation & Emergency Airway Support in the Pediatric E...
GEMC- Rapid Sequence Intubation & Emergency Airway Support in the Pediatric E...
 
GEMC- Ocular Emgercencies- Resident Training
GEMC- Ocular Emgercencies- Resident TrainingGEMC- Ocular Emgercencies- Resident Training
GEMC- Ocular Emgercencies- Resident Training
 
GEMC- Disorders of the Pleura, Mediastinum, and Chest Wall- Resident Training
GEMC- Disorders of the Pleura, Mediastinum, and Chest Wall- Resident TrainingGEMC- Disorders of the Pleura, Mediastinum, and Chest Wall- Resident Training
GEMC- Disorders of the Pleura, Mediastinum, and Chest Wall- Resident Training
 
GEMC- Dental Emergencies and Common Dental Blocks- Resident Training
GEMC- Dental Emergencies and Common Dental Blocks- Resident TrainingGEMC- Dental Emergencies and Common Dental Blocks- Resident Training
GEMC- Dental Emergencies and Common Dental Blocks- Resident Training
 
GEMC- EMedHome Board Review: Procedures- Resident Training
GEMC- EMedHome Board Review: Procedures- Resident TrainingGEMC- EMedHome Board Review: Procedures- Resident Training
GEMC- EMedHome Board Review: Procedures- Resident Training
 
GEMC- Arthritis and Arthrocentesis- Resident Training
GEMC- Arthritis and Arthrocentesis- Resident TrainingGEMC- Arthritis and Arthrocentesis- Resident Training
GEMC- Arthritis and Arthrocentesis- Resident Training
 
GEMC- Bursitis, Tendonitis, Fibromyalgia, and RSD- Resident Training
GEMC- Bursitis, Tendonitis, Fibromyalgia, and RSD- Resident TrainingGEMC- Bursitis, Tendonitis, Fibromyalgia, and RSD- Resident Training
GEMC- Bursitis, Tendonitis, Fibromyalgia, and RSD- Resident Training
 
GEMC- Right Upper Quadrant Ultrasound- Resident Training
GEMC- Right Upper Quadrant Ultrasound- Resident TrainingGEMC- Right Upper Quadrant Ultrasound- Resident Training
GEMC- Right Upper Quadrant Ultrasound- Resident Training
 
GEMC- Cardiovascular Board Review Session 3- Resident Training
GEMC- Cardiovascular Board Review Session 3- Resident TrainingGEMC- Cardiovascular Board Review Session 3- Resident Training
GEMC- Cardiovascular Board Review Session 3- Resident Training
 
GEMC- Cardiovascular Board Review Session 2- Resident Training
GEMC- Cardiovascular Board Review Session 2- Resident TrainingGEMC- Cardiovascular Board Review Session 2- Resident Training
GEMC- Cardiovascular Board Review Session 2- Resident Training
 
GEMC- Cardiovascular Board Review Session 1- Resident Training
GEMC- Cardiovascular Board Review Session 1- Resident TrainingGEMC- Cardiovascular Board Review Session 1- Resident Training
GEMC- Cardiovascular Board Review Session 1- Resident Training
 
GEMC: Nursing Process and Linkage between Theory and Practice
GEMC: Nursing Process and Linkage between Theory and PracticeGEMC: Nursing Process and Linkage between Theory and Practice
GEMC: Nursing Process and Linkage between Theory and Practice
 
2014 gemc-nursing-lapham-general survey and patient care management
2014 gemc-nursing-lapham-general survey and patient care management2014 gemc-nursing-lapham-general survey and patient care management
2014 gemc-nursing-lapham-general survey and patient care management
 
GEMC: The Role of Radiography in the Initial Evaluation of C-Spine Trauma
GEMC: The Role of Radiography in the Initial Evaluation of C-Spine TraumaGEMC: The Role of Radiography in the Initial Evaluation of C-Spine Trauma
GEMC: The Role of Radiography in the Initial Evaluation of C-Spine Trauma
 
GEMC - Mammal and Human Bite Injuries
GEMC - Mammal and Human Bite InjuriesGEMC - Mammal and Human Bite Injuries
GEMC - Mammal and Human Bite Injuries
 
GEMC- Sickle Cell Disease: Special Considerations in Pediatrics- Resident Tra...
GEMC- Sickle Cell Disease: Special Considerations in Pediatrics- Resident Tra...GEMC- Sickle Cell Disease: Special Considerations in Pediatrics- Resident Tra...
GEMC- Sickle Cell Disease: Special Considerations in Pediatrics- Resident Tra...
 
GEMC- Ghana Grab Bag Pediatric Quiz- Resident Training
GEMC- Ghana Grab Bag Pediatric Quiz- Resident TrainingGEMC- Ghana Grab Bag Pediatric Quiz- Resident Training
GEMC- Ghana Grab Bag Pediatric Quiz- Resident Training
 

Recently uploaded

Beyond the EU: DORA and NIS 2 Directive's Global Impact
Beyond the EU: DORA and NIS 2 Directive's Global ImpactBeyond the EU: DORA and NIS 2 Directive's Global Impact
Beyond the EU: DORA and NIS 2 Directive's Global ImpactPECB
 
Advanced Views - Calendar View in Odoo 17
Advanced Views - Calendar View in Odoo 17Advanced Views - Calendar View in Odoo 17
Advanced Views - Calendar View in Odoo 17Celine George
 
Presentation by Andreas Schleicher Tackling the School Absenteeism Crisis 30 ...
Presentation by Andreas Schleicher Tackling the School Absenteeism Crisis 30 ...Presentation by Andreas Schleicher Tackling the School Absenteeism Crisis 30 ...
Presentation by Andreas Schleicher Tackling the School Absenteeism Crisis 30 ...EduSkills OECD
 
Key note speaker Neum_Admir Softic_ENG.pdf
Key note speaker Neum_Admir Softic_ENG.pdfKey note speaker Neum_Admir Softic_ENG.pdf
Key note speaker Neum_Admir Softic_ENG.pdfAdmir Softic
 
Grant Readiness 101 TechSoup and Remy Consulting
Grant Readiness 101 TechSoup and Remy ConsultingGrant Readiness 101 TechSoup and Remy Consulting
Grant Readiness 101 TechSoup and Remy ConsultingTechSoup
 
Nutritional Needs Presentation - HLTH 104
Nutritional Needs Presentation - HLTH 104Nutritional Needs Presentation - HLTH 104
Nutritional Needs Presentation - HLTH 104misteraugie
 
Sports & Fitness Value Added Course FY..
Sports & Fitness Value Added Course FY..Sports & Fitness Value Added Course FY..
Sports & Fitness Value Added Course FY..Disha Kariya
 
Activity 01 - Artificial Culture (1).pdf
Activity 01 - Artificial Culture (1).pdfActivity 01 - Artificial Culture (1).pdf
Activity 01 - Artificial Culture (1).pdfciinovamais
 
Sanyam Choudhary Chemistry practical.pdf
Sanyam Choudhary Chemistry practical.pdfSanyam Choudhary Chemistry practical.pdf
Sanyam Choudhary Chemistry practical.pdfsanyamsingh5019
 
Explore beautiful and ugly buildings. Mathematics helps us create beautiful d...
Explore beautiful and ugly buildings. Mathematics helps us create beautiful d...Explore beautiful and ugly buildings. Mathematics helps us create beautiful d...
Explore beautiful and ugly buildings. Mathematics helps us create beautiful d...christianmathematics
 
Russian Escort Service in Delhi 11k Hotel Foreigner Russian Call Girls in Delhi
Russian Escort Service in Delhi 11k Hotel Foreigner Russian Call Girls in DelhiRussian Escort Service in Delhi 11k Hotel Foreigner Russian Call Girls in Delhi
Russian Escort Service in Delhi 11k Hotel Foreigner Russian Call Girls in Delhikauryashika82
 
Measures of Dispersion and Variability: Range, QD, AD and SD
Measures of Dispersion and Variability: Range, QD, AD and SDMeasures of Dispersion and Variability: Range, QD, AD and SD
Measures of Dispersion and Variability: Range, QD, AD and SDThiyagu K
 
Software Engineering Methodologies (overview)
Software Engineering Methodologies (overview)Software Engineering Methodologies (overview)
Software Engineering Methodologies (overview)eniolaolutunde
 
social pharmacy d-pharm 1st year by Pragati K. Mahajan
social pharmacy d-pharm 1st year by Pragati K. Mahajansocial pharmacy d-pharm 1st year by Pragati K. Mahajan
social pharmacy d-pharm 1st year by Pragati K. Mahajanpragatimahajan3
 
Z Score,T Score, Percential Rank and Box Plot Graph
Z Score,T Score, Percential Rank and Box Plot GraphZ Score,T Score, Percential Rank and Box Plot Graph
Z Score,T Score, Percential Rank and Box Plot GraphThiyagu K
 
Call Girls in Dwarka Mor Delhi Contact Us 9654467111
Call Girls in Dwarka Mor Delhi Contact Us 9654467111Call Girls in Dwarka Mor Delhi Contact Us 9654467111
Call Girls in Dwarka Mor Delhi Contact Us 9654467111Sapana Sha
 
IGNOU MSCCFT and PGDCFT Exam Question Pattern: MCFT003 Counselling and Family...
IGNOU MSCCFT and PGDCFT Exam Question Pattern: MCFT003 Counselling and Family...IGNOU MSCCFT and PGDCFT Exam Question Pattern: MCFT003 Counselling and Family...
IGNOU MSCCFT and PGDCFT Exam Question Pattern: MCFT003 Counselling and Family...PsychoTech Services
 

Recently uploaded (20)

Beyond the EU: DORA and NIS 2 Directive's Global Impact
Beyond the EU: DORA and NIS 2 Directive's Global ImpactBeyond the EU: DORA and NIS 2 Directive's Global Impact
Beyond the EU: DORA and NIS 2 Directive's Global Impact
 
Advanced Views - Calendar View in Odoo 17
Advanced Views - Calendar View in Odoo 17Advanced Views - Calendar View in Odoo 17
Advanced Views - Calendar View in Odoo 17
 
Presentation by Andreas Schleicher Tackling the School Absenteeism Crisis 30 ...
Presentation by Andreas Schleicher Tackling the School Absenteeism Crisis 30 ...Presentation by Andreas Schleicher Tackling the School Absenteeism Crisis 30 ...
Presentation by Andreas Schleicher Tackling the School Absenteeism Crisis 30 ...
 
Advance Mobile Application Development class 07
Advance Mobile Application Development class 07Advance Mobile Application Development class 07
Advance Mobile Application Development class 07
 
Key note speaker Neum_Admir Softic_ENG.pdf
Key note speaker Neum_Admir Softic_ENG.pdfKey note speaker Neum_Admir Softic_ENG.pdf
Key note speaker Neum_Admir Softic_ENG.pdf
 
Grant Readiness 101 TechSoup and Remy Consulting
Grant Readiness 101 TechSoup and Remy ConsultingGrant Readiness 101 TechSoup and Remy Consulting
Grant Readiness 101 TechSoup and Remy Consulting
 
Nutritional Needs Presentation - HLTH 104
Nutritional Needs Presentation - HLTH 104Nutritional Needs Presentation - HLTH 104
Nutritional Needs Presentation - HLTH 104
 
Sports & Fitness Value Added Course FY..
Sports & Fitness Value Added Course FY..Sports & Fitness Value Added Course FY..
Sports & Fitness Value Added Course FY..
 
Activity 01 - Artificial Culture (1).pdf
Activity 01 - Artificial Culture (1).pdfActivity 01 - Artificial Culture (1).pdf
Activity 01 - Artificial Culture (1).pdf
 
Código Creativo y Arte de Software | Unidad 1
Código Creativo y Arte de Software | Unidad 1Código Creativo y Arte de Software | Unidad 1
Código Creativo y Arte de Software | Unidad 1
 
Sanyam Choudhary Chemistry practical.pdf
Sanyam Choudhary Chemistry practical.pdfSanyam Choudhary Chemistry practical.pdf
Sanyam Choudhary Chemistry practical.pdf
 
Explore beautiful and ugly buildings. Mathematics helps us create beautiful d...
Explore beautiful and ugly buildings. Mathematics helps us create beautiful d...Explore beautiful and ugly buildings. Mathematics helps us create beautiful d...
Explore beautiful and ugly buildings. Mathematics helps us create beautiful d...
 
Russian Escort Service in Delhi 11k Hotel Foreigner Russian Call Girls in Delhi
Russian Escort Service in Delhi 11k Hotel Foreigner Russian Call Girls in DelhiRussian Escort Service in Delhi 11k Hotel Foreigner Russian Call Girls in Delhi
Russian Escort Service in Delhi 11k Hotel Foreigner Russian Call Girls in Delhi
 
Mattingly "AI & Prompt Design: Structured Data, Assistants, & RAG"
Mattingly "AI & Prompt Design: Structured Data, Assistants, & RAG"Mattingly "AI & Prompt Design: Structured Data, Assistants, & RAG"
Mattingly "AI & Prompt Design: Structured Data, Assistants, & RAG"
 
Measures of Dispersion and Variability: Range, QD, AD and SD
Measures of Dispersion and Variability: Range, QD, AD and SDMeasures of Dispersion and Variability: Range, QD, AD and SD
Measures of Dispersion and Variability: Range, QD, AD and SD
 
Software Engineering Methodologies (overview)
Software Engineering Methodologies (overview)Software Engineering Methodologies (overview)
Software Engineering Methodologies (overview)
 
social pharmacy d-pharm 1st year by Pragati K. Mahajan
social pharmacy d-pharm 1st year by Pragati K. Mahajansocial pharmacy d-pharm 1st year by Pragati K. Mahajan
social pharmacy d-pharm 1st year by Pragati K. Mahajan
 
Z Score,T Score, Percential Rank and Box Plot Graph
Z Score,T Score, Percential Rank and Box Plot GraphZ Score,T Score, Percential Rank and Box Plot Graph
Z Score,T Score, Percential Rank and Box Plot Graph
 
Call Girls in Dwarka Mor Delhi Contact Us 9654467111
Call Girls in Dwarka Mor Delhi Contact Us 9654467111Call Girls in Dwarka Mor Delhi Contact Us 9654467111
Call Girls in Dwarka Mor Delhi Contact Us 9654467111
 
IGNOU MSCCFT and PGDCFT Exam Question Pattern: MCFT003 Counselling and Family...
IGNOU MSCCFT and PGDCFT Exam Question Pattern: MCFT003 Counselling and Family...IGNOU MSCCFT and PGDCFT Exam Question Pattern: MCFT003 Counselling and Family...
IGNOU MSCCFT and PGDCFT Exam Question Pattern: MCFT003 Counselling and Family...
 

Problem Solving Sessions

  • 1. Author(s): Rebecca W. Van Dyke, M.D., 2012 License: Unless otherwise noted, this material is made available under the terms of the Creative Commons Attribution – Share Alike 3.0 License: http://creativecommons.org/licenses/by-sa/3.0/ We have reviewed this material in accordance with U.S. Copyright Law and have tried to maximize your ability to use, share, and adapt it. The citation key on the following slide provides information about how you may share and adapt this material. Copyright holders of content included in this material should contact open.michigan@umich.edu with any questions, corrections, or clarification regarding the use of content. For more information about how to cite these materials visit http://open.umich.edu/education/about/terms-of-use. Any medical information in this material is intended to inform and educate and is not a tool for self-diagnosis or a replacement for medical evaluation, advice, diagnosis or treatment by a healthcare professional. Please speak to your physician if you have questions about your medical condition. Viewer discretion is advised: Some medical content is graphic and may not be suitable for all viewers.
  • 2. Attribution Key for more information see: http://open.umich.edu/wiki/AttributionPolicy Use + Share + Adapt { Content the copyright holder, author, or law permits you to use, share and adapt. } Public Domain – Government: Works that are produced by the U.S. Government. (17 USC § 105) Public Domain – Expired: Works that are no longer protected due to an expired copyright term. Public Domain – Self Dedicated: Works that a copyright holder has dedicated to the public domain. Creative Commons – Zero Waiver Creative Commons – Attribution License Creative Commons – Attribution Share Alike License Creative Commons – Attribution Noncommercial License Creative Commons – Attribution Noncommercial Share Alike License GNU – Free Documentation License Make Your Own Assessment { Content Open.Michigan believes can be used, shared, and adapted because it is ineligible for copyright. } Public Domain – Ineligible: Works that are ineligible for copyright protection in the U.S. (17 USC § 102(b)) *laws in your jurisdiction may differ { Content Open.Michigan has used under a Fair Use determination. } Fair Use: Use of works that is determined to be Fair consistent with the U.S. Copyright Act. (17 USC § 107) *laws in your jurisdiction may differ Our determination DOES NOT mean that all uses of this 3rd-party content are Fair Uses and we DO NOT guarantee that your use of the content is Fair. To use this content you should do your own independent analysis to determine whether or not your use will be Fair.
  • 4. Learning Objectives • After attending one or more these eleven 30 minute sessions the student should be able to: • • 1. Demonstrate increased competence in using knowledge obtained from lectures and textbooks to answer patient questions about their diseases. • • 2. Demonstrate increased competence at correctly interpreting laboratory tests for liver disease, viral hepatitis, analysis of stools samples and analysis of ascites samples. • • 3. Demonstrate increased competence in identifying abnormalities on radiographic studies and suggesting a diagnosis (or diagnoses). • • 4. Demonstrate increased competence in selecting drug treatments for GERD, diarrhea. • • 5. Demonstrate increased competence in identifying what complications might occur when patients undergo certain GI surgical procedures and how these may be managed. • • 6. Demonstrate increased problem-solving skills for patients with GI diseases.
  • 5. Industry Relationship Disclosures Industry Supported Research and Outside Relationships • None
  • 7. Your patient has an endoscopy and these pictures were obtained. What problems might this patient have or develop in the future? Why did this occur?
  • 8. Case A • A 24 year old medical student has developed epigastric pain. • She thinks she has an ulcer. • How would you determine whether she has an ulcer? • Is there a “best” approach?
  • 9. Case – B A 75 year old patient comes to see you as a new patient in geriatrics clinic. In taking a history you discover he had surgery for recurrent stomach ulcers in 1963. He thinks part of his stomach was removed at the time. What kind of operation did he likely have? What is his current anatomy likely to be? What problems can occur with these types of surgery?
  • 10. Case C • 75 year old woman is concerned she may have gastric cancer. • This disease arises in the gastric epithelium. • How would you try to find out if she has gastric cancer? • What can you do? • Is there a “best”way to answer her question?
  • 11. Case - D • The family is gathered for a Super Bowl party, complete with all the usual munchies. • Your uncle pulls you aside and tells you he gets bad heartburn from the salsa, which he loves. • Now that you are a medical student, he want advice on how to prevent the heartburn. • What do you suggest?
  • 12. Case – D1 • Three hours later your cousin, who gorged on pizza, complains of terrible heartburn and wants you to do something RIGHT NOW to relieve her symptoms. • What do you suggest?
  • 13. Case – D2 • Your grandmother overhears these conversations and loudly complains that her doctor has told her she is susceptible to stomach ulcers and therefore she cannot take her “arthritis” pills. • She has severe pain in her hands, hip and knees and wants to know why the doctor took away her medications and what you can do to solve her problem. • What pills were removed and why? • What options are available?
  • 14. Case E • A 60 year old woman went to the Mayo Clinic and was told she had Zollinger-Ellison syndrome. • She returns to Ann Arbor and comes to you for care. • You recall the Z-E syndrome is due to a small gastrin-secreting tumor. • What problems might you expect her to develop? • What signs and symptoms might she develop? • What could you do to help treat or prevent these problems?
  • 15. Case F • A patient comes to see you having been told he has a duodenal ulcer. • He wants to know how/why he got the ulcer. • What do you tell him (patient education)? • What can he do to heal this ulcer as fast as possible (treatment)? • What can he do to prevent future ulcers (secondary prevention)?
  • 17. • A 29 year old man come to see you because of recurrent gas and diarrhea. • He wants to know: • Does he have lactose intolerance? • Does milk cause his symptoms? • Does he have lactase deficiency? • How would you answer each question for him?
  • 18. Case – G A 55 year old man comes to your office for evaluation of diarrhea. Diarrhea began in the past year although he cannot pinpoint an exact time. He notes 3-5 loose stools during the day and none at night. He has no abdominal pain and has not lost weight. His only other medical problem is frequent heartburn for which he takes antacids. The physical examination is normal except the digital rectal examination which yields loose/watery light brown stool that is negative for occult blood. What is your differential diagnosis? What do you do next?
  • 19. Case – G-2 Diarrhea continues after he stops the antacid intake Stool electrolytes: Na = 90 mEq/l K = 40 Cl = 40 Stool/plasma osmolality = 295 Stool osmotic gap = ???? Diagnostic possibilities?
  • 20. Analysis of Fecal Electrolytes – normal values Sodium ~20-40 mEq/l Potassium ~90 Chloride ~15 HCO3- ~30 Anions (SO4-2, ~85 PO4-3, fatty acids) Magnesium ~10-20 Volume 0.2-0.4 liters/day Plasma osmolality ~290-300 mEq/l Stool osmolality ~290-300 mEq/l Osmotic gap ~40-70 Fecal pH > ~5.4
  • 21. Analysis of Fecal Electrolytes - I Sodium 105 Potassium 30 Chloride 69 Magnesium 15 Electrolyte pattern? Volume 3 liters/day Plasma osmolality 295 mEq/l Stool osmolality 301 mEq/l Osmotic gap? Fecal pH > 5.4
  • 22. Analysis of Fecal Electrolytes - II Sodium 22 Potassium 26 Chloride 55 Electrolyte pattern? Volume 1.3 liters/day Plasma osmolality 295 mEq/l Stool osmolality 299 mEq/l Osmotic gap? Fecal pH > 5.4
  • 23. Analysis of Fecal Electrolytes - III Sodium 43 Potassium 89 Chloride 18 Magnesium 18 Electrolyte pattern? Volume 0.28 liters/day Plasma osmolality 295 mEq/l Stool osmolality 302 mEq/l Osmotic gap?
  • 24. Analysis of Fecal Electrolytes - IV Sodium 32 Potassium 28 Chloride 10 Electrolyte pattern? Volume 1.5 liters/day Plasma osmolality 295 mEq/l Osmotic gap?
  • 25. Analysis of Fecal Electrolytes - V Sodium 20 Potassium 45 Chloride 10 Magnesium 10 Electrolyte pattern? Volume 1.1 liters/day Plasma osmolality 295 mEq/l Stool osmolality 140 mEq/l Osmotic gap?
  • 26. Analysis of Fecal Electrolytes - VI Sodium 103 Potassium 42 Chloride 18 Magnesium 11 Electrolyte pattern? Volume 1.8 liters/day Plasma osmolality 295 mEq/l Stool osmolality 303 mEq/l Osmotic gap?
  • 27. Analysis of Fecal Electrolytes – secretory diarrhea Sodium 105 Potassium 30 Chloride 69 Magnesium 15 Electrolyte pattern? Volume 3 liters/day Plasma osmolality 295 mEq/l Stool osmolality 301 mEq/l Osmotic gap? Fecal pH > 5.4 High sodium, no osmotic gap, normal pH, high volume = secretory diarrhea
  • 29. Case – H I An 85 year old woman with untreated atrial fibrillation throws an embolus that lodges in the superior mesenteric artery proximal to the origin of the ileocolic artery causing ischemic necrosis of what part of the bowel?
  • 30. Case – H II 200 cm ileum
  • 31. Case – H III She undergoes emergency surgery with resection of 200cm of this part of the bowel and anastamosis of the proximal ileum to the transverse colon. She recovers uneventfully,however what problems might she develop due to loss of this bowel?
  • 32. Case – I- I • A patient has systemic sclerosis (scleroderma). • You read in your textbook that this disease destroys GI smooth muscle, first in the esophagus, later small bowel and colon. • The patient wants to know what problems she is likely to develop in the future. • What do you tell her?
  • 33. Case J • A patient travels to South America and is bitten by the reduviid bug, transmitting Trypanosoma cruzi. • He develops Chagas disease. • You recall your parasitology and that this trypanosome specifically involves the wall of the esophagus and LES destroying NO-secreting neurons. • What signs and symptoms will the patient develop and why?
  • 34. Case K • A patient comes to you requesting an injection of botulinus toxin in the lower esophageal sphincter. • What will this toxin do in this location? • What type of symptoms might be expected to be relieved? • What complications might occur?
  • 35. Case L A 75 year old man comes to see you because of diarrhea. He has 5-7 loose watery stools a day and urgency. He says the volume of the stools are moderate. You evaluate him and find on biopsy that he has microscopic colitis. You remember that this is a chronic disorder that cannot be cured. What treatment options are available? What would you advise him to do?
  • 36. Case M A 28 year old woman comes to see you because of urgency, diarrhea and bright red blood mixed with some of her stools. This has been going on for about 2 months. You evaluate her and make a diagnosis of left- sided ulcerative colitis. What problems need management? What types of treatment might you give her?
  • 37. Case N A 35 year old woman comes to you for evaluation of diarrhea and weight loss. Last year she had surgery for active Crohn’s disease at which time over a meter (>100 cm) of distal ileum was resected. She recovered well from the surgery but then developed frequent soft stools and has lost about 20 pounds of weight. What pathophysiologic explanation can you develop for her diarrhea and how would you test your hypothesis? What other problem(s) would you look for in this patient?
  • 38. Case – O- I • A patient comes to see you saying a doctor told him he had “colitis”. • He has daily soft or liquid stools and thinks he was told he had ulcerative colitis, but is not sure. • He occasionally sees streaks of blood. • Symptoms have been present for 6 months. • What diagnoses are possible? • How could you determine the type of “colitis”?
  • 39. Case – O- II • The same patient returns with medical records including a colonoscopy report. • The colonoscopy showed inflammation and ulcers located only in the sigmoid colon and in the cecum. • What do you think the correct diagnosis is? • How would you prove this? • He continues to have symptoms - how would you treat him?
  • 41. Case P • A 30 year old business woman has worsening diarrhea, now 3-6 times a day, especially after meals. None at night • She had these symptoms for several years but they have worsened lately as business is worse. • Every few weeks she gets constipation for 3-4 days, then diarrhea. • She gets crampy abdominal pains on many days. • What problems could cause her symptoms? • How would you evaluate and treat her?
  • 42. Case – Q- I An 18 year old girl presents with an 8 week history of mild mid- abdominal pain, diarrhea, and weight loss. The pain is described as “achy” but not very specific. She notices it more at night when she is trying to sleep. She now has 3-5 soft semi-formed bowel movements per day and occasionally she has to get up at night to pass stool. She notes anorexia and “loss of energy” and has lost 5 pounds in weight. She has not seen blood in her stools. She has not traveled in the last year and knows no one with diarrhea. She takes no medications. What is your differential diagnosis? What do you do next? Any lab tests you want?
  • 43. Case – Q- II WBC 10,000 (nl 4,000-8,000) Hct 32 (nl 36-45) MCV 95 (nl 80-99) Platelets 250,000 (nl 150,000-350,000) Electrolytes, BUN, Cr normal What have you learned? What can you think of to do next?
  • 44. What do these images show? What signs or symptoms might this patient have had?
  • 45. Case R • A patient is brought into the ER having vomited bright red blood 6 times this morning, each time “cups and cups” of blood. • What do you do first? • What are the diagnostic possibilities?
  • 46. Case S You have a patient in whom food does not pass out of the stomach normally. What symptoms do you expect? What underlying diseases might the patient have?
  • 48. Case T-1 A 43 year old woman is brought to the emergency room because of vomiting blood. Yesterday she felt somewhat weak, nauseated and not her usual self. This morning she began vomiting, brought up large amounts of bright red blood and clots and felt dizzy. She has been taking ibuprofen for the past three weeks because of shoulder bursitis. She has a long history of taking antacids for burning epigastric and substernal pain that occurs 3-5 times per week between meals or at night. Four years ago when she tried to donate blood, she was told she could not do so because she had abnormal liver tests. She does not smoke and no longer drinks alcohol although she drank regularly until she was 35. What is her main problem when you see her in the ER? What do you want to do next?
  • 49. Case T - 2 On physical examination she looks reasonably well. Lying: BP 110/60, P85 Sitting: BP 90/45, P110 Examination is normal. She has active bowel sounds, but epigastric tenderness upon palpation without any masses. Stool obtained by digital rectal examination is burgundy/black, soft and markedly positive by hemoccult testing. How much has this woman bled? What else do you want to know?
  • 50. Case T - 3 WBC 7.5 (nl ≤ 10) Hct. 22 (nl 36 - 45) Platelets 200,000 (nl 150,000 - 350,000) PT 12 sec (nl 9 - 12.5 sec) What does this tell you? What do you do next?
  • 51. Case U A 35 year old man comes to see you for a general medical examination. When you take a history, you find out that his paternal grandfather, father’s brother and his older brother all have colon cancer. What are your concerns about your patient? What more information might you want to get from the history? What do you advise your patient to do and why?
  • 52. Case V • A patient comes to the ER with severe and frequent nausea and vomiting of food and gastric contents after virtually every meal for the past 6 weeks. • She has lost 15 pounds • On exam she is thin with orthostatic changes in BP and pulse. • What electrolyte/blood test abnormalities might you expect? • What types of problems could cause these symptoms? • How do you prove what is causing her problem?
  • 53. How do you educate and treat each of these patients? Cases W 1-3 • Your 11 year old future Olympic ice skater gets nauseated and often vomits after her fast spins. • Your pregnant sister starts vomiting every morning. • A patient with lung cancer tells you he cannot continue chemotherapy as his vomiting is too severe.
  • 55. Case X • A 45 year old woman comes to see you as she has started passing gas and stool through her vagina. • Needless to say she is very distressed and wants this solved immediately. • What had to have happened? • How can you prove it? • Later this week, think about what kind of diseases could have caused this problem.
  • 56. What is abnormal on this x-ray and what is it?
  • 57. What do you see on this x-ray?
  • 58. What does this x-ray show? What would you find on exam?
  • 59. What do you see in this image – the abdominal x-ray?
  • 60. CT enterography of normal abdomen for your interest
  • 61. Case Y • A patient has a “lupus anticoagulant” or anti-phospholipid antibody and develops a portal vein thrombosis with complete obstruction of portal venous blood flow. • What complications would you expect to occur?
  • 63. Case 1 Laboratory Findings Bilirubin 8.5 (0.2-1.2 mg/dl) Alkaline Phos. 250 (23-100 IU/ml) AST 1500 (20-35 IU/ml) ALT 1750 (18-30 IU/ml) Albumin 4.0 (3.5-4.5 g/dl) PT 11.0 (10.5-12.0 sec)
  • 64. Case 2 Laboratory Findings Bilirubin 8.5 (0.2-1.2 mg/dl) Alkaline Phos. 675 (23-100 IU/ml) AST 92 (20-35 IU/ml) ALT 99 (18-30 IU/ml) Albumin 4.0 (3.5-4.5 g/dl) PT 11.5 (10.5-12.0 sec)
  • 65. Case 3 Laboratory Findings Bilirubin 3.5 (0.2-1.2 mg/dl) Alkaline Phos. 190 (23-100 IU/ml) AST 300 (20-35 IU/ml) ALT 400 (18-30 IU/ml) Albumin 4.0 (3.5-4.5 g/dl) PT 12.0 (10.5-12.0 sec)
  • 66. Case 4 Laboratory Findings Bilirubin 0.8 (0.2-1.2 mg/dl) Alkaline Phos. 90 (23-100 IU/ml) AST 2500 (20-35 IU/ml) ALT 28 (18-30 IU/ml) Albumin 4.0 (3.5-4.5 g/dl) PT 11.0 (10.5-12.0 sec)
  • 67. Case 5 Laboratory Findings Bilirubin 9.0 (0.2-1.2 mg/dl) Alkaline Phos. 175 (23-100 IU/ml) AST 210 (20-35 IU/ml) ALT 100 (18-30 IU/ml) Albumin 3.2 (3.5-4.5 g/dl) PT 14.5 (10.5-12.0 sec)
  • 68. Case 6 Laboratory Findings Bilirubin 9.0 (0.2-1.2 mg/dl) Alkaline Phos. 200 (23-100 IU/ml) AST 2500 (20-35 IU/ml) ALT 3200 (18-30 IU/ml) Albumin 4.0 (3.5-4.5 g/dl) PT 14.5 (10.5-12.0 sec)
  • 69. Case 7 Laboratory Findings Bilirubin 1.0 (0.2-1.2 mg/dl) Alkaline Phos. 555 (23-100 IU/ml) AST 20 (20-35 IU/ml) ALT 22 (18-30 IU/ml) Albumin 4.0 (3.5-4.5 g/dl) PT 11.5 (10.5-12.0 sec)
  • 70. Case 8 Laboratory Findings Bilirubin 3.0 (0.2-1.2 mg/dl) Alkaline Phos. 120 (23-100 IU/ml) AST 65 (20-35 IU/ml) ALT 68 (18-30 IU/ml) Albumin 2.0 (3.5-4.5 g/dl) PT 15.5 (10.5-12.0 sec)
  • 72. Analysis of Serologic Tests for Viral Hepatitis – Case Z • A 23 year old medical student comes to the emergency room with the following symptoms: – 1 week of nausea, vomiting, severe fatigue and 1 day of jaundice • Lab tests are: – bilirubin: 5.6 (nl <1.1 mg/dl) – AST/ALT 1500/1900 (nl<70 IU) – Alk Phos 330 (nl<110 IU) – Prothrombin time 11.5 sec (nl<12 sec) • You send every serologic test you can think of – the results come back and you have to interpret them.
  • 73. Analysis of Hepatitis Tests Hepatitis A positive IgM antibody negative IgG antibody Hepatitis B negative sAg (surface antigen) positive sAB (surface antibody) negative IgM antibody to core negative IgG antibody to core negative eAg (e antigen) negative eAB (e antibody) negative DNA Hepatitis C negative antibody (IgG) negative RNA by PCR Hepatitis D negative RNA by PCR negative IgM antibody negative IgG antibody Hepatitis E negative IgM antibody negative IgG antibody
  • 74. Analysis of Hepatitis Tests Hepatitis A negative IgM antibody negative IgG antibody Hepatitis B negative sAg (surface antigen) positive sAB (surface antibody) negative IgM antibody to core positive IgG antibody to core negative eAg (e antigen) negative eAB (e antibody) negative DNA Hepatitis C negative antibody (IgG) positive RNA by PCR Hepatitis D negative RNA by PCR negative IgM antibody negative IgG antibody Hepatitis E negative IgM antibody negative IgG antibody
  • 75. Analysis of Hepatitis Tests Hepatitis A negative IgM antibody positive IgG antibody Hepatitis B negative sAg (surface antigen) negative IgM antibody to core negative IgG antibody to core negative eAg (e antigen) negative eAB (e antibody) negative DNA Hepatitis C negative antibody (IgG) negative RNA by PCR Hepatitis D negative RNA by PCR negative IgM antibody negative IgG antibody Hepatitis E positive IgM antibody negative IgG antibody
  • 76. Analysis of Hepatitis Tests Hepatitis A negative IgM antibody positive IgG antibody Hepatitis B positive sAg (surface antigen) negative sAB (surface antibody) positive IgM antibody to core negative IgG antibody to core positive eAg (e antigen) negative eAB (e antibody) positive DNA Hepatitis C negative antibody (IgG) negative RNA by PCR Hepatitis D negative RNA by PCR negative IgM antibody negative IgG antibody Hepatitis E negative IgM antibody negative IgG antibody
  • 77. Analysis of Hepatitis Tests Hepatitis A negative IgM antibody positive IgG antibody Hepatitis B positive sAg (surface antigen) negative sAB (surface antibody) negative IgM antibody to core positive IgG antibody to core negative eAg (e antigen) positive eAB (e antibody) negative DNA Hepatitis C negative antibody (IgG) negative RNA by PCR Hepatitis D positive RNA by PCR positive IgM antibody negative IgG antibody Hepatitis E negative IgM antibody negative IgG antibody
  • 78. Analysis of Hepatitis Tests Hepatitis A negative IgM antibody negative IgG antibody Hepatitis B negative sAg (surface antigen) negative sAB (surface antibody) negative IgM antibody to core negative IgG antibody to core negative eAg (e antigen) negative eAB (e antibody) negative DNA Hepatitis C positive antibody (IgG) positive RNA by PCR Hepatitis D negative RNA by PCR negative IgM antibody negative IgG antibody Hepatitis E positive IgM antibody negative IgG antibody
  • 79. Case AA • A 32 year old man had developed a greatly enlarged abdomen over the past several months. • He asks you what this is due to. • What are the possible causes of his enlarged abdomen? • What can you do to investigate the cause of this enlargement?
  • 80. A patient brings you a liver biopsy. Here is one image from it. What does the patient have and what problems might the patient develop?
  • 81. Case BB - A patient comes to see you with liver biopsy slides. This is what you see. What is the problem and what symptoms might the patient develop? H&E stain Trichrome stain
  • 83. Case CC • A patient with cirrhosis wants to know if she has the following. How would you try to find out? – Hepatocellular carcinoma – Ascites – Esophageal varices – Hepatic hydrothorax • pleural effusion due to ascites
  • 84. Case DD • A 35 year old woman had a CT scan of the abdomen that showed a thrombus occluding her splenic vein. • She wants to know what problems she will develop because of this. • Why did she develop this thrombus?
  • 85. What are these skin findings?
  • 86. Case EE • A patient with cirrhosis and portal hypertension is concerned about his health and reads advertisements for Gator-aid - that if makes you stronger and better able to exercise. He buys a large case and drinks 3-4 bottles per day. • What problem(s) is he likely to develop?
  • 87. Case FF • A patient with cirrhosis and portal hypertension is concerned that his arms and legs are getting thinner. • He starts to work out at a gym and consults the nutritionist there who advises a high protein diet with meat and protein powder supplements. • What problem might he develop?
  • 88. What do you see here? What diagnoses are you thinking of?
  • 89. Case GG A patient is admitted to your service for evaluation of new onset ascites. You perform a paracentesis and send the fluid for a variety of tests. The tests come back and your resident asks you to interpret the results.
  • 90. • Remember Dr. Moseley’s lecture on complications of cirrhosis and ascites • When a patient presents with new onset ascites, diagnostic studies need to be done and analyzed to determine the cause of ascites and whether there is a complication (infection)
  • 91.
  • 92. Ascites Fluid Analysis Test Ascites Serum (normal) Total protein 2.2 7.5 (8-9g/dl) Albumin 1.0 2.9 (3.5-5.0g/dl) Cell count (PMNs) 10 4000 (2000-6000) Triglycerides 15 150 (<160mg/dl) Amylase 20 40 (0-130 U/l) How would you interpret these results?
  • 93. - Peritonitis Non-cirrhotic portal hypertension “Hydrodynamic/transudative” “Exudative”
  • 94. Ascites Fluid Analysis Test Ascites Serum Total protein 6.5 7.5 (8-9g/dl) Albumin 3.5 4.3 (3.5-5.0g/dl) Cell count (PMNs) 10 4000 (2000-6000) Triglycerides 15 150 (<160mg/dl) Amylase 20 40 (0-130 U/l) How would you interpret these results? What more could you do?
  • 95. Ascites Fluid Analysis Test Ascites Serum Total protein 7.4 7.5 (8-9g/dl) Albumin 3.4 4.1 (3.5-5.0g/dl) Cell count (PMNs) 10 4000 (2000-6000) Triglycerides 15 150 (<160mg/dl) Amylase 950 40 (0-130 U/l) How would you interpret these results?
  • 96. Ascites Fluid Analysis Test Ascites Serum Total protein 6.5 7.5 (8-9g/dl) Albumin 3.6 4.5 (3.5-5.0g/dl) Cell count (PMNs) 10 4000 (2000-6000) Triglycerides 820 150 (<160mg/dl) Amylase 20 40 (0-130 U/l) How would you interpret these results?
  • 97. Ascites Fluid Analysis Test Ascites Serum (normal) Total protein 3.5 7.5 (8-9g/dl) Albumin 2.5 4.5 (3.5-5.0g/dl) Cell count (PMNs) 10 4000 (2000-6000) Triglycerides 15 150 (<160mg/dl) Amylase 20 40 (0-130 U/l) How would you interpret these results?
  • 98. Ascites Fluid Analysis Test Ascites Serum (normal) Total protein 2.6 7.6 (8-9g/dl) Albumin 1.0 2.8 (3.5-5.0g/dl) Cell count (PMNs) 774 8300 (2000-6000) Triglycerides 15 150 (<160mg/dl) Amylase 20 40 (0-130 U/l) How would you interpret these results?
  • 99. Ascites Fluid Analysis Test Ascites Serum (normal) Total protein 6.8 7.6 (8-9g/dl) Albumin 3.9 4.3 (3.5-5.0g/dl) Cell count (PMNs) 10,200 18,950 (2000-6000) Triglycerides 15 150 (<160mg/dl) Amylase 20 40 (0-130 U/l) How would you interpret these results? What other tests might be abnormal?
  • 100. Ascites Fluid Analysis Test Ascites Serum (normal) Total protein 6.8 7.6 (8-9 g/dl) Albumin 3.9 4.3 (3.5-5.0 g/dl) Cell count (PMNs) 10,200 18,950 (2000-6000) Triglycerides 15 142 (<150 mg/dl) Amylase 20 40 (0-130 U/l) LDH 495 150 (120-240 IU/l) Glucose 15 112 (73-110 mg/dl)
  • 102. Acute hepatitis B case from small group yesterday • What is this patient’s prognosis (what may happen to him)? • He has an infectious disease – what do you need to do when wearing your public health hat?
  • 103. Case HH • A patient of yours takes a drug (A) that is metabolized in the liver primarily by CYP3A4 • She develops a new problem and you give her another drug (B). However it is also metabolized in the liver by CYP3A4 • What problems could occur?
  • 104. Case II • You are treating a patient for tuberculosis with isoniazid. You read that this is primarily metabolized by CYP2E1. • Your patient tells you he drinks alcohol regularily - up to 6 beers per day plus some liquor on the weekends. • Does this concern you?
  • 105. Case JJ A 35 year old man presents with 18 hours of pain that began in the periumbilical region as an “aching” pain. Subsequently, the pain became more severe and was felt also in the right lower quadrant. He has vomited stomach contents several times and feels nauseated. Examination shows a pale sweaty man curled on his side in pain. Pulse is 110 and he is febrile (T 100.8˚F). Bowel sounds are absent. He is very tender in the right lower quadrant and “guards” in this area. When the examiner pushes down on the left lower quadrant and releases quickly, severe pain is felt in the right lower quadrant. Laboratory studies are essentially normal except for a leukocytosis (WBC 13,000). What is your differential diagnosis?
  • 106. Case KK - 1: This patient has epigastric pain and nausea/vomiting. What do you see?
  • 107. Case LL • A patient develops localized carcinoma of the ampulla of Vater. • This is treated by surgical resection of the duodenum and part of the pancreas. The stomach and biliary/pancreatic ducts are each anastamosed to loops of jejunum. • What problems might develop?
  • 108. Patient Case MM • 65 year old comes in with left lower quadrant abdominal aching pain for the last 2 days with fever • He is concerned he may have acute diverticulitis as he has had similar symptoms in the past. • How would you determine whether he had diverticulitis?
  • 110. Case NN • A patient has an abdominal x-ray performed for an unrelated reason. • The radiologist reports the presence of calcifications throughout the pancreas • The patient wants to know what this means? • What do you tell her? • What might you want to do with her?
  • 111. Case OO • You are caring for a patient with acute pancreatitis • The patient asks you what complications he might develop – what do you tell him? • He wants to know how he got this disease – what do you tell him?
  • 112. Case PP-1 • You are taking care of a patient with acute pancreatitis and the resident asks you how sick the patient is. • What factors can you assess to tell if this patient is severely ill with pancreatitis or not?
  • 113. Case QQ • A 20 year old girl was involved in a severe MVA. She was driving and had a crush injury to her pancreas from the steering wheel. • She develops severe acute pancreatitis and requires surgery to resect the necrotic tissue and she loses about 80% of her pancreas. • She survives but is likely to develop what complications?
  • 114. Case RR Three patients are admitted to your service- 1. one has nausea, vomiting and dehydration due to gastric outlet obstruction, 2. one is jaundiced, and 3. one has severe epigastric pain and weight loss. What single disease could they all have?
  • 115. Case SS • A patient comes to see you with 6 months of chronic abdominal pain thought to be due to chronic pancreatitis (from alcohol). • Upon evaluation you determine the patient has lost about 35 pounds of weight over this period and has malabsorption due to pancreatic insufficiency. • Why did the patient develop pancreatic insufficiency? • How can you treat the pain? • How can you treat the pancreatic insufficiency?
  • 116. Case TT - 1 A 35 year old man is brought to the emergency room with a one day history of abdominal pain. The patient felt well until 18 hours ago when he developed constant epigastric pain that he describes as “boring through” to the back associated with nausea and vomiting of gastric contents. He feels somewhat better if he sits up bent forward. On examination he is pale, sweaty and in pain. Vital signs are normal except a pulse of 100. Abdominal exam shows hypoactive infrequent bowel sounds and severe epigastric pain on even mild palpation. There is no “rebound” tenderness. What is your differential diagnosis? What do you do next?
  • 117. Case TT - 2 WBC 14,000 (nl ≤ 10) Hct 45 (nl 36 - 45) Platelets 250,000 (nl 150,000 - 350,000) Elecrolytes, Bun, Cr normal AST 145 I.U. (nl < 45) ALT 160 I.U. (nl < 50) Alk phos 290 I.U. (nl < 110) Bili 2.0 mg/dl (nl < 1.0) Albumin 4.5 gm/dl (nl 3.5-4.5) PT 10.5 sec (nl 10-12.5) Amylase 10 X normal Lipase 20 X normal Abdominal x-ray shows no free air
  • 118. Case UU • A patient with pancreatic cancer develops jaundice. • Why did the jaundice develop? • What complications will occur? • What can be done about the jaundice?
  • 120. What do you see in this x-ray?
  • 121. What do you see here? What diseases could cause this? What accompanying problems might you expect?
  • 122. Case UU A patient comes to see you as he had an x-ray done when he went to the Emergency Room for abdominal pain and it showed gallstones. He wants to know if he should have his gallbladder taken out. Who should have their gallbladder removed? What signs and symptoms might he have had that would convince you he should have a cholecystectomy?
  • 123. VV - Each of the following patients has gallstones – what type? • 45 year old woman who is obese and has had 4 children. • A 21 year old man with sickle cell disease. • A 25 year old woman with Crohn’s disease who has had >100 cm of her terminal ileum resected. • A 50 year old who lost most of his small bowel due to a gunshot wound and receives nutrition through a central vein (total parenteral nutrition).
  • 124. What do you see here?
  • 125. 75 year old man with diabetes and history of acute right upper quadrant pain, fever, elevated white blood count. This is emphysematous cholecystitis. Cultures of the gallbladder grew Clostridium perfringens Air in GB, air/fluid level Air in the wall of the GB
  • 126. Case VV A 35 year old man presents with 12 hours of pain that began as a constant “aching”, “sharp” mid abdominal pain that slowly moved to the right upper quadrant. The patient is tachycardic (P=98) with a low grade fever (100˚F) and on abdominal examination has moderate pain on palpation in the periumbilical/epigastric region and severe pain in the right upper quadrant. Laboratory studies are normal except for a mild leukocytosis (WBC 11,000). What is your differential diagnosis? What do you do next?
  • 127. Case WW • A patient has pain in the epigastric region that began 2 days ago. What organs could be reponsible for this pain? • What do each of the following pain patterns suggest as etiologies? – Sharp/severe penetrating pain radiating through to the back. – Dull aching pain that increased over several hours and then moved to the RUQ? – Burning epigastric pain that did not radiate?